Proving $(X,Y)$ is a normal vector when $Xsim N(1,1)$ and $Ymid Xsim N(3X,4)$












1














Suppose I have a random vector $(X,Y)$ with $Xsimmathcal{N}(1,1)$ and $Y|X = x simmathcal{N}(3x,4)$.



I need to prove that $(X,Y)$ is a normal vector as well.



To do that I want to explicitly write the vector of expected values and the $2x2$ matrix of variance and covariance.



I know that the first entry of the vector of expected values is 1 and the entry $C_{1,1}$ of the matrix is 1 as well. However I am struggling to see how I can derive the density function of Y from the conditional one.



Any suggestions?










share|cite|improve this question
























  • Joint density of $(X,Y)$ is $f_{X,Y}(x,y)=f_{Ymid X}(y)f_X(x)=...$
    – StubbornAtom
    Jan 4 at 15:07










  • Yes, but I am trying to calculate E[Y], V[Y] and Cov[X,Y] to "fill" the matrices. I am struggling with Cov[X,Y], specifically with E[XY]. Do you have an hint for that?
    – qcc101
    Jan 4 at 15:10










  • Comment made by @StubbornAtom below his/her answer needs to be emphasized. Just finding the mean vector and the variance-covariance matrix do not show the joint-normality of $X$ and $Y$.
    – Just_to_Answer
    Jan 4 at 20:15


















1














Suppose I have a random vector $(X,Y)$ with $Xsimmathcal{N}(1,1)$ and $Y|X = x simmathcal{N}(3x,4)$.



I need to prove that $(X,Y)$ is a normal vector as well.



To do that I want to explicitly write the vector of expected values and the $2x2$ matrix of variance and covariance.



I know that the first entry of the vector of expected values is 1 and the entry $C_{1,1}$ of the matrix is 1 as well. However I am struggling to see how I can derive the density function of Y from the conditional one.



Any suggestions?










share|cite|improve this question
























  • Joint density of $(X,Y)$ is $f_{X,Y}(x,y)=f_{Ymid X}(y)f_X(x)=...$
    – StubbornAtom
    Jan 4 at 15:07










  • Yes, but I am trying to calculate E[Y], V[Y] and Cov[X,Y] to "fill" the matrices. I am struggling with Cov[X,Y], specifically with E[XY]. Do you have an hint for that?
    – qcc101
    Jan 4 at 15:10










  • Comment made by @StubbornAtom below his/her answer needs to be emphasized. Just finding the mean vector and the variance-covariance matrix do not show the joint-normality of $X$ and $Y$.
    – Just_to_Answer
    Jan 4 at 20:15
















1












1








1


1





Suppose I have a random vector $(X,Y)$ with $Xsimmathcal{N}(1,1)$ and $Y|X = x simmathcal{N}(3x,4)$.



I need to prove that $(X,Y)$ is a normal vector as well.



To do that I want to explicitly write the vector of expected values and the $2x2$ matrix of variance and covariance.



I know that the first entry of the vector of expected values is 1 and the entry $C_{1,1}$ of the matrix is 1 as well. However I am struggling to see how I can derive the density function of Y from the conditional one.



Any suggestions?










share|cite|improve this question















Suppose I have a random vector $(X,Y)$ with $Xsimmathcal{N}(1,1)$ and $Y|X = x simmathcal{N}(3x,4)$.



I need to prove that $(X,Y)$ is a normal vector as well.



To do that I want to explicitly write the vector of expected values and the $2x2$ matrix of variance and covariance.



I know that the first entry of the vector of expected values is 1 and the entry $C_{1,1}$ of the matrix is 1 as well. However I am struggling to see how I can derive the density function of Y from the conditional one.



Any suggestions?







probability normal-distribution conditional-probability bivariate-distributions






share|cite|improve this question















share|cite|improve this question













share|cite|improve this question




share|cite|improve this question








edited Jan 4 at 19:02









StubbornAtom

5,39411138




5,39411138










asked Jan 4 at 14:48









qcc101qcc101

477113




477113












  • Joint density of $(X,Y)$ is $f_{X,Y}(x,y)=f_{Ymid X}(y)f_X(x)=...$
    – StubbornAtom
    Jan 4 at 15:07










  • Yes, but I am trying to calculate E[Y], V[Y] and Cov[X,Y] to "fill" the matrices. I am struggling with Cov[X,Y], specifically with E[XY]. Do you have an hint for that?
    – qcc101
    Jan 4 at 15:10










  • Comment made by @StubbornAtom below his/her answer needs to be emphasized. Just finding the mean vector and the variance-covariance matrix do not show the joint-normality of $X$ and $Y$.
    – Just_to_Answer
    Jan 4 at 20:15




















  • Joint density of $(X,Y)$ is $f_{X,Y}(x,y)=f_{Ymid X}(y)f_X(x)=...$
    – StubbornAtom
    Jan 4 at 15:07










  • Yes, but I am trying to calculate E[Y], V[Y] and Cov[X,Y] to "fill" the matrices. I am struggling with Cov[X,Y], specifically with E[XY]. Do you have an hint for that?
    – qcc101
    Jan 4 at 15:10










  • Comment made by @StubbornAtom below his/her answer needs to be emphasized. Just finding the mean vector and the variance-covariance matrix do not show the joint-normality of $X$ and $Y$.
    – Just_to_Answer
    Jan 4 at 20:15


















Joint density of $(X,Y)$ is $f_{X,Y}(x,y)=f_{Ymid X}(y)f_X(x)=...$
– StubbornAtom
Jan 4 at 15:07




Joint density of $(X,Y)$ is $f_{X,Y}(x,y)=f_{Ymid X}(y)f_X(x)=...$
– StubbornAtom
Jan 4 at 15:07












Yes, but I am trying to calculate E[Y], V[Y] and Cov[X,Y] to "fill" the matrices. I am struggling with Cov[X,Y], specifically with E[XY]. Do you have an hint for that?
– qcc101
Jan 4 at 15:10




Yes, but I am trying to calculate E[Y], V[Y] and Cov[X,Y] to "fill" the matrices. I am struggling with Cov[X,Y], specifically with E[XY]. Do you have an hint for that?
– qcc101
Jan 4 at 15:10












Comment made by @StubbornAtom below his/her answer needs to be emphasized. Just finding the mean vector and the variance-covariance matrix do not show the joint-normality of $X$ and $Y$.
– Just_to_Answer
Jan 4 at 20:15






Comment made by @StubbornAtom below his/her answer needs to be emphasized. Just finding the mean vector and the variance-covariance matrix do not show the joint-normality of $X$ and $Y$.
– Just_to_Answer
Jan 4 at 20:15












2 Answers
2






active

oldest

votes


















2














You could say $Y=3X+Z$ where $Zsim N(0,4)$ independent of $X$.



It is then an easy step to say that $Ysim N(3times 1+0,3^2times 1+4)$ and




  • $E[X]=1$

  • $E[Y]=3$

  • $text{Var}(X)=1$

  • $text{Var}(Y)=13$

  • $text{Cov}(X,Y)=text{Cov}(X,3X)=3text{Var}(X)=3$






share|cite|improve this answer





















  • This is the direction I took as well. However I did not use an additional Z. Therefore I am struggling with calculating the covariance between X and Y, could you take a look at my comment to the answer below?
    – qcc101
    Jan 4 at 15:28



















2














You can proceed with moment generating functions.



Joint MGF of $(X,Y)$ is



begin{align}
M(s,t)&=Eleft[e^{sX+tY}right]
\&=Eleft[E(e^{sX+tY}mid X)right]
\&=Eleft[e^{sX}E(e^{tY}mid X)right]
end{align}



From the given information, you should be able to show that this MGF corresponds to the MGF of a bivariate normal distribution. That would complete your proof using the uniqueness property of MGF.






share|cite|improve this answer





















  • I am not sure I am following you. In my problem I did find Variance and Expected value of Y, using the tower property of conditioning. I stil need to find the two symmetric entries of the matrix of variance and covariance to complete what I want to do. That is I need to find Covariance[X,Y]. Calculating, I get to this: $$Cov[X,Y] = E[XY] - 3E[X^2] -E[Y] + 3E[X]$$ Above, I know everything except E[XY]. I did the following: $ E[XY] = E[ E[XY|X]] = E[YE[X|X]] = E[Y E[X]]$ but I get the wrong result ( covariance should be 3 but I get -3). Could you point out what I did wrong?
    – qcc101
    Jan 4 at 15:26










  • @qcc101 $E(XY)=E(E(XYmid X))=E(XE(Ymid X))=...$
    – StubbornAtom
    Jan 4 at 15:29










  • Right, silly mistake. I will investigate the part about moment generating functions as well. Thank you.
    – qcc101
    Jan 4 at 15:30






  • 1




    @qcc101 Without the argument that $X$ and $Y$ are linear combinations of independent normal variables and hence $(X,Y)$ is bivariate normal (as seen in the answer by @Henry), I don't think finding the variance-covariance matrix (only) as you are trying to do is sufficient for the given question.
    – StubbornAtom
    Jan 4 at 15:37













Your Answer





StackExchange.ifUsing("editor", function () {
return StackExchange.using("mathjaxEditing", function () {
StackExchange.MarkdownEditor.creationCallbacks.add(function (editor, postfix) {
StackExchange.mathjaxEditing.prepareWmdForMathJax(editor, postfix, [["$", "$"], ["\\(","\\)"]]);
});
});
}, "mathjax-editing");

StackExchange.ready(function() {
var channelOptions = {
tags: "".split(" "),
id: "69"
};
initTagRenderer("".split(" "), "".split(" "), channelOptions);

StackExchange.using("externalEditor", function() {
// Have to fire editor after snippets, if snippets enabled
if (StackExchange.settings.snippets.snippetsEnabled) {
StackExchange.using("snippets", function() {
createEditor();
});
}
else {
createEditor();
}
});

function createEditor() {
StackExchange.prepareEditor({
heartbeatType: 'answer',
autoActivateHeartbeat: false,
convertImagesToLinks: true,
noModals: true,
showLowRepImageUploadWarning: true,
reputationToPostImages: 10,
bindNavPrevention: true,
postfix: "",
imageUploader: {
brandingHtml: "Powered by u003ca class="icon-imgur-white" href="https://imgur.com/"u003eu003c/au003e",
contentPolicyHtml: "User contributions licensed under u003ca href="https://creativecommons.org/licenses/by-sa/3.0/"u003ecc by-sa 3.0 with attribution requiredu003c/au003e u003ca href="https://stackoverflow.com/legal/content-policy"u003e(content policy)u003c/au003e",
allowUrls: true
},
noCode: true, onDemand: true,
discardSelector: ".discard-answer"
,immediatelyShowMarkdownHelp:true
});


}
});














draft saved

draft discarded


















StackExchange.ready(
function () {
StackExchange.openid.initPostLogin('.new-post-login', 'https%3a%2f%2fmath.stackexchange.com%2fquestions%2f3061724%2fproving-x-y-is-a-normal-vector-when-x-sim-n1-1-and-y-mid-x-sim-n3x-4%23new-answer', 'question_page');
}
);

Post as a guest















Required, but never shown

























2 Answers
2






active

oldest

votes








2 Answers
2






active

oldest

votes









active

oldest

votes






active

oldest

votes









2














You could say $Y=3X+Z$ where $Zsim N(0,4)$ independent of $X$.



It is then an easy step to say that $Ysim N(3times 1+0,3^2times 1+4)$ and




  • $E[X]=1$

  • $E[Y]=3$

  • $text{Var}(X)=1$

  • $text{Var}(Y)=13$

  • $text{Cov}(X,Y)=text{Cov}(X,3X)=3text{Var}(X)=3$






share|cite|improve this answer





















  • This is the direction I took as well. However I did not use an additional Z. Therefore I am struggling with calculating the covariance between X and Y, could you take a look at my comment to the answer below?
    – qcc101
    Jan 4 at 15:28
















2














You could say $Y=3X+Z$ where $Zsim N(0,4)$ independent of $X$.



It is then an easy step to say that $Ysim N(3times 1+0,3^2times 1+4)$ and




  • $E[X]=1$

  • $E[Y]=3$

  • $text{Var}(X)=1$

  • $text{Var}(Y)=13$

  • $text{Cov}(X,Y)=text{Cov}(X,3X)=3text{Var}(X)=3$






share|cite|improve this answer





















  • This is the direction I took as well. However I did not use an additional Z. Therefore I am struggling with calculating the covariance between X and Y, could you take a look at my comment to the answer below?
    – qcc101
    Jan 4 at 15:28














2












2








2






You could say $Y=3X+Z$ where $Zsim N(0,4)$ independent of $X$.



It is then an easy step to say that $Ysim N(3times 1+0,3^2times 1+4)$ and




  • $E[X]=1$

  • $E[Y]=3$

  • $text{Var}(X)=1$

  • $text{Var}(Y)=13$

  • $text{Cov}(X,Y)=text{Cov}(X,3X)=3text{Var}(X)=3$






share|cite|improve this answer












You could say $Y=3X+Z$ where $Zsim N(0,4)$ independent of $X$.



It is then an easy step to say that $Ysim N(3times 1+0,3^2times 1+4)$ and




  • $E[X]=1$

  • $E[Y]=3$

  • $text{Var}(X)=1$

  • $text{Var}(Y)=13$

  • $text{Cov}(X,Y)=text{Cov}(X,3X)=3text{Var}(X)=3$







share|cite|improve this answer












share|cite|improve this answer



share|cite|improve this answer










answered Jan 4 at 15:26









HenryHenry

98.4k475162




98.4k475162












  • This is the direction I took as well. However I did not use an additional Z. Therefore I am struggling with calculating the covariance between X and Y, could you take a look at my comment to the answer below?
    – qcc101
    Jan 4 at 15:28


















  • This is the direction I took as well. However I did not use an additional Z. Therefore I am struggling with calculating the covariance between X and Y, could you take a look at my comment to the answer below?
    – qcc101
    Jan 4 at 15:28
















This is the direction I took as well. However I did not use an additional Z. Therefore I am struggling with calculating the covariance between X and Y, could you take a look at my comment to the answer below?
– qcc101
Jan 4 at 15:28




This is the direction I took as well. However I did not use an additional Z. Therefore I am struggling with calculating the covariance between X and Y, could you take a look at my comment to the answer below?
– qcc101
Jan 4 at 15:28











2














You can proceed with moment generating functions.



Joint MGF of $(X,Y)$ is



begin{align}
M(s,t)&=Eleft[e^{sX+tY}right]
\&=Eleft[E(e^{sX+tY}mid X)right]
\&=Eleft[e^{sX}E(e^{tY}mid X)right]
end{align}



From the given information, you should be able to show that this MGF corresponds to the MGF of a bivariate normal distribution. That would complete your proof using the uniqueness property of MGF.






share|cite|improve this answer





















  • I am not sure I am following you. In my problem I did find Variance and Expected value of Y, using the tower property of conditioning. I stil need to find the two symmetric entries of the matrix of variance and covariance to complete what I want to do. That is I need to find Covariance[X,Y]. Calculating, I get to this: $$Cov[X,Y] = E[XY] - 3E[X^2] -E[Y] + 3E[X]$$ Above, I know everything except E[XY]. I did the following: $ E[XY] = E[ E[XY|X]] = E[YE[X|X]] = E[Y E[X]]$ but I get the wrong result ( covariance should be 3 but I get -3). Could you point out what I did wrong?
    – qcc101
    Jan 4 at 15:26










  • @qcc101 $E(XY)=E(E(XYmid X))=E(XE(Ymid X))=...$
    – StubbornAtom
    Jan 4 at 15:29










  • Right, silly mistake. I will investigate the part about moment generating functions as well. Thank you.
    – qcc101
    Jan 4 at 15:30






  • 1




    @qcc101 Without the argument that $X$ and $Y$ are linear combinations of independent normal variables and hence $(X,Y)$ is bivariate normal (as seen in the answer by @Henry), I don't think finding the variance-covariance matrix (only) as you are trying to do is sufficient for the given question.
    – StubbornAtom
    Jan 4 at 15:37


















2














You can proceed with moment generating functions.



Joint MGF of $(X,Y)$ is



begin{align}
M(s,t)&=Eleft[e^{sX+tY}right]
\&=Eleft[E(e^{sX+tY}mid X)right]
\&=Eleft[e^{sX}E(e^{tY}mid X)right]
end{align}



From the given information, you should be able to show that this MGF corresponds to the MGF of a bivariate normal distribution. That would complete your proof using the uniqueness property of MGF.






share|cite|improve this answer





















  • I am not sure I am following you. In my problem I did find Variance and Expected value of Y, using the tower property of conditioning. I stil need to find the two symmetric entries of the matrix of variance and covariance to complete what I want to do. That is I need to find Covariance[X,Y]. Calculating, I get to this: $$Cov[X,Y] = E[XY] - 3E[X^2] -E[Y] + 3E[X]$$ Above, I know everything except E[XY]. I did the following: $ E[XY] = E[ E[XY|X]] = E[YE[X|X]] = E[Y E[X]]$ but I get the wrong result ( covariance should be 3 but I get -3). Could you point out what I did wrong?
    – qcc101
    Jan 4 at 15:26










  • @qcc101 $E(XY)=E(E(XYmid X))=E(XE(Ymid X))=...$
    – StubbornAtom
    Jan 4 at 15:29










  • Right, silly mistake. I will investigate the part about moment generating functions as well. Thank you.
    – qcc101
    Jan 4 at 15:30






  • 1




    @qcc101 Without the argument that $X$ and $Y$ are linear combinations of independent normal variables and hence $(X,Y)$ is bivariate normal (as seen in the answer by @Henry), I don't think finding the variance-covariance matrix (only) as you are trying to do is sufficient for the given question.
    – StubbornAtom
    Jan 4 at 15:37
















2












2








2






You can proceed with moment generating functions.



Joint MGF of $(X,Y)$ is



begin{align}
M(s,t)&=Eleft[e^{sX+tY}right]
\&=Eleft[E(e^{sX+tY}mid X)right]
\&=Eleft[e^{sX}E(e^{tY}mid X)right]
end{align}



From the given information, you should be able to show that this MGF corresponds to the MGF of a bivariate normal distribution. That would complete your proof using the uniqueness property of MGF.






share|cite|improve this answer












You can proceed with moment generating functions.



Joint MGF of $(X,Y)$ is



begin{align}
M(s,t)&=Eleft[e^{sX+tY}right]
\&=Eleft[E(e^{sX+tY}mid X)right]
\&=Eleft[e^{sX}E(e^{tY}mid X)right]
end{align}



From the given information, you should be able to show that this MGF corresponds to the MGF of a bivariate normal distribution. That would complete your proof using the uniqueness property of MGF.







share|cite|improve this answer












share|cite|improve this answer



share|cite|improve this answer










answered Jan 4 at 15:17









StubbornAtomStubbornAtom

5,39411138




5,39411138












  • I am not sure I am following you. In my problem I did find Variance and Expected value of Y, using the tower property of conditioning. I stil need to find the two symmetric entries of the matrix of variance and covariance to complete what I want to do. That is I need to find Covariance[X,Y]. Calculating, I get to this: $$Cov[X,Y] = E[XY] - 3E[X^2] -E[Y] + 3E[X]$$ Above, I know everything except E[XY]. I did the following: $ E[XY] = E[ E[XY|X]] = E[YE[X|X]] = E[Y E[X]]$ but I get the wrong result ( covariance should be 3 but I get -3). Could you point out what I did wrong?
    – qcc101
    Jan 4 at 15:26










  • @qcc101 $E(XY)=E(E(XYmid X))=E(XE(Ymid X))=...$
    – StubbornAtom
    Jan 4 at 15:29










  • Right, silly mistake. I will investigate the part about moment generating functions as well. Thank you.
    – qcc101
    Jan 4 at 15:30






  • 1




    @qcc101 Without the argument that $X$ and $Y$ are linear combinations of independent normal variables and hence $(X,Y)$ is bivariate normal (as seen in the answer by @Henry), I don't think finding the variance-covariance matrix (only) as you are trying to do is sufficient for the given question.
    – StubbornAtom
    Jan 4 at 15:37




















  • I am not sure I am following you. In my problem I did find Variance and Expected value of Y, using the tower property of conditioning. I stil need to find the two symmetric entries of the matrix of variance and covariance to complete what I want to do. That is I need to find Covariance[X,Y]. Calculating, I get to this: $$Cov[X,Y] = E[XY] - 3E[X^2] -E[Y] + 3E[X]$$ Above, I know everything except E[XY]. I did the following: $ E[XY] = E[ E[XY|X]] = E[YE[X|X]] = E[Y E[X]]$ but I get the wrong result ( covariance should be 3 but I get -3). Could you point out what I did wrong?
    – qcc101
    Jan 4 at 15:26










  • @qcc101 $E(XY)=E(E(XYmid X))=E(XE(Ymid X))=...$
    – StubbornAtom
    Jan 4 at 15:29










  • Right, silly mistake. I will investigate the part about moment generating functions as well. Thank you.
    – qcc101
    Jan 4 at 15:30






  • 1




    @qcc101 Without the argument that $X$ and $Y$ are linear combinations of independent normal variables and hence $(X,Y)$ is bivariate normal (as seen in the answer by @Henry), I don't think finding the variance-covariance matrix (only) as you are trying to do is sufficient for the given question.
    – StubbornAtom
    Jan 4 at 15:37


















I am not sure I am following you. In my problem I did find Variance and Expected value of Y, using the tower property of conditioning. I stil need to find the two symmetric entries of the matrix of variance and covariance to complete what I want to do. That is I need to find Covariance[X,Y]. Calculating, I get to this: $$Cov[X,Y] = E[XY] - 3E[X^2] -E[Y] + 3E[X]$$ Above, I know everything except E[XY]. I did the following: $ E[XY] = E[ E[XY|X]] = E[YE[X|X]] = E[Y E[X]]$ but I get the wrong result ( covariance should be 3 but I get -3). Could you point out what I did wrong?
– qcc101
Jan 4 at 15:26




I am not sure I am following you. In my problem I did find Variance and Expected value of Y, using the tower property of conditioning. I stil need to find the two symmetric entries of the matrix of variance and covariance to complete what I want to do. That is I need to find Covariance[X,Y]. Calculating, I get to this: $$Cov[X,Y] = E[XY] - 3E[X^2] -E[Y] + 3E[X]$$ Above, I know everything except E[XY]. I did the following: $ E[XY] = E[ E[XY|X]] = E[YE[X|X]] = E[Y E[X]]$ but I get the wrong result ( covariance should be 3 but I get -3). Could you point out what I did wrong?
– qcc101
Jan 4 at 15:26












@qcc101 $E(XY)=E(E(XYmid X))=E(XE(Ymid X))=...$
– StubbornAtom
Jan 4 at 15:29




@qcc101 $E(XY)=E(E(XYmid X))=E(XE(Ymid X))=...$
– StubbornAtom
Jan 4 at 15:29












Right, silly mistake. I will investigate the part about moment generating functions as well. Thank you.
– qcc101
Jan 4 at 15:30




Right, silly mistake. I will investigate the part about moment generating functions as well. Thank you.
– qcc101
Jan 4 at 15:30




1




1




@qcc101 Without the argument that $X$ and $Y$ are linear combinations of independent normal variables and hence $(X,Y)$ is bivariate normal (as seen in the answer by @Henry), I don't think finding the variance-covariance matrix (only) as you are trying to do is sufficient for the given question.
– StubbornAtom
Jan 4 at 15:37






@qcc101 Without the argument that $X$ and $Y$ are linear combinations of independent normal variables and hence $(X,Y)$ is bivariate normal (as seen in the answer by @Henry), I don't think finding the variance-covariance matrix (only) as you are trying to do is sufficient for the given question.
– StubbornAtom
Jan 4 at 15:37




















draft saved

draft discarded




















































Thanks for contributing an answer to Mathematics Stack Exchange!


  • Please be sure to answer the question. Provide details and share your research!

But avoid



  • Asking for help, clarification, or responding to other answers.

  • Making statements based on opinion; back them up with references or personal experience.


Use MathJax to format equations. MathJax reference.


To learn more, see our tips on writing great answers.





Some of your past answers have not been well-received, and you're in danger of being blocked from answering.


Please pay close attention to the following guidance:


  • Please be sure to answer the question. Provide details and share your research!

But avoid



  • Asking for help, clarification, or responding to other answers.

  • Making statements based on opinion; back them up with references or personal experience.


To learn more, see our tips on writing great answers.




draft saved


draft discarded














StackExchange.ready(
function () {
StackExchange.openid.initPostLogin('.new-post-login', 'https%3a%2f%2fmath.stackexchange.com%2fquestions%2f3061724%2fproving-x-y-is-a-normal-vector-when-x-sim-n1-1-and-y-mid-x-sim-n3x-4%23new-answer', 'question_page');
}
);

Post as a guest















Required, but never shown





















































Required, but never shown














Required, but never shown












Required, but never shown







Required, but never shown

































Required, but never shown














Required, but never shown












Required, but never shown







Required, but never shown







Popular posts from this blog

An IMO inspired problem

Management

Has there ever been an instance of an active nuclear power plant within or near a war zone?